Showing the closure of a compact subset need not be compactIsn't every subset of a compact space compact?Example on closure of a subset of a subspace of a topological space in Munkres's TopologyCompact subset of a non compact topological spaceWhy subspace of a compact space not compactIs $mathbbR$ compact under the co-countable and co-finite topologies?Show that $mathbbQ$ is not locally compact with a characterization of local compactnessIs the closure of a compact set compact?A topological space is locally compact then here is an open base at each point has all of its set with compact closure$BbbR^omega$ is not Locally CompactShowing a subset of $Bbb R^2$ is compact with the relative topology

Finding files for which a command fails

Why doesn't Newton's third law mean a person bounces back to where they started when they hit the ground?

I’m planning on buying a laser printer but concerned about the life cycle of toner in the machine

How do you conduct xenoanthropology after first contact?

Modification to Chariots for Heavy Cavalry Analogue for 4-armed race

Download, install and reboot computer at night if needed

Can an x86 CPU running in real mode be considered to be basically an 8086 CPU?

If Manufacturer spice model and Datasheet give different values which should I use?

What typically incentivizes a professor to change jobs to a lower ranking university?

cryptic clue: mammal sounds like relative consumer (8)

What is the meaning of "of trouble" in the following sentence?

How to make payment on the internet without leaving a money trail?

A newer friend of my brother's gave him a load of baseball cards that are supposedly extremely valuable. Is this a scam?

How to determine if window is maximised or minimised from bash script

Closed subgroups of abelian groups

Patience, young "Padovan"

Calculus Optimization - Point on graph closest to given point

How is it possible for user's password to be changed after storage was encrypted? (on OS X, Android)

Why did the Germans forbid the possession of pet pigeons in Rostov-on-Don in 1941?

What do you call something that goes against the spirit of the law, but is legal when interpreting the law to the letter?

How old can references or sources in a thesis be?

Where to refill my bottle in India?

Could a US political party gain complete control over the government by removing checks & balances?

My colleague's body is amazing



Showing the closure of a compact subset need not be compact


Isn't every subset of a compact space compact?Example on closure of a subset of a subspace of a topological space in Munkres's TopologyCompact subset of a non compact topological spaceWhy subspace of a compact space not compactIs $mathbbR$ compact under the co-countable and co-finite topologies?Show that $mathbbQ$ is not locally compact with a characterization of local compactnessIs the closure of a compact set compact?A topological space is locally compact then here is an open base at each point has all of its set with compact closure$BbbR^omega$ is not Locally CompactShowing a subset of $Bbb R^2$ is compact with the relative topology













1












$begingroup$


Could someone tell me if my line of reasoning is correct here:



Say we have the topological space $(mathbbN, T)$ comprising of the empty set together with all subsets of $Bbb N$ that contain the element $1$.



I want to show that the closure of a compact set in this topology need not be compact.



Let $A equiv 1, ...., n$.
Then, this set is compact as it can be covered by a single open set in $T$.
Its limit points are every number in $Bbb N$ which is not 1.
Therefore, its closure is $overlineA = Bbb N$



If the above is true then I get confused because it seems that $Bbb N$ is an open set in $T$ so therefore can it not also be covered with a single open set in T and so would be compact as well? (I know intuitively that compact, being in some sense a measure of 'smallness', would indicate that $Bbb N$ shouldn't be compact but I don't see how to get that line of reasoning using the properties of this topology)










share|cite|improve this question











$endgroup$







  • 3




    $begingroup$
    Your reasoning for $A$ being compact seems incorrect. $A = 1, ldots, n$ is compact because given an open cover $ B_alpha $ of $A$, we can find $B_alpha_k ni k$ for each $k in A$ so that $ B_alpha_k _k=1^n$ covers $A$. $mathbbN$ is not compact in your topology because the infinite cover $ 1, ldots, n _n=1^infty$ of $mathbbN$ cannot be reduce to a finite subcover.
    $endgroup$
    – parsiad
    4 hours ago











  • $begingroup$
    @parsiad Ah okay , so I should think of each one of the points in A needing to be covered by an open set in T rather than the set itself being able to be fully covered by a single set. So then using that fact I could then use the rest of my argument to say that $Bbb N$ is not compact as given an open cover $B_alpha$ of $Bbb N$ then for each $nin Bbb N$ we can cover it with a set from T but we need infinitely many to do it for all points so it's not compact ?
    $endgroup$
    – can'tcauchy
    4 hours ago






  • 1




    $begingroup$
    I'm not sure what you mean, but you should use this definition of compactness. I've included an argument for why $mathbbN$ is not compact in my comment above that you can revisit after understanding the linked definition.
    $endgroup$
    – parsiad
    4 hours ago











  • $begingroup$
    @parsiad what you said in your comment is what I meant. Thanks ! But maybe the part I was unclear about was that I think I had been considering the set A=1,...n as being able to be covered by the set 1,....n in T, but as it says in the definition we have to think about A as being covered by points in a collection of open sets within A, and this can be reduced to a finite number of open sets which do this . Are the limit point and closure part of my argument is fine ,correct ?
    $endgroup$
    – can'tcauchy
    4 hours ago











  • $begingroup$
    Yes, I think that part of your argument is correct. I posted a detailed answer anyways.
    $endgroup$
    – parsiad
    4 hours ago
















1












$begingroup$


Could someone tell me if my line of reasoning is correct here:



Say we have the topological space $(mathbbN, T)$ comprising of the empty set together with all subsets of $Bbb N$ that contain the element $1$.



I want to show that the closure of a compact set in this topology need not be compact.



Let $A equiv 1, ...., n$.
Then, this set is compact as it can be covered by a single open set in $T$.
Its limit points are every number in $Bbb N$ which is not 1.
Therefore, its closure is $overlineA = Bbb N$



If the above is true then I get confused because it seems that $Bbb N$ is an open set in $T$ so therefore can it not also be covered with a single open set in T and so would be compact as well? (I know intuitively that compact, being in some sense a measure of 'smallness', would indicate that $Bbb N$ shouldn't be compact but I don't see how to get that line of reasoning using the properties of this topology)










share|cite|improve this question











$endgroup$







  • 3




    $begingroup$
    Your reasoning for $A$ being compact seems incorrect. $A = 1, ldots, n$ is compact because given an open cover $ B_alpha $ of $A$, we can find $B_alpha_k ni k$ for each $k in A$ so that $ B_alpha_k _k=1^n$ covers $A$. $mathbbN$ is not compact in your topology because the infinite cover $ 1, ldots, n _n=1^infty$ of $mathbbN$ cannot be reduce to a finite subcover.
    $endgroup$
    – parsiad
    4 hours ago











  • $begingroup$
    @parsiad Ah okay , so I should think of each one of the points in A needing to be covered by an open set in T rather than the set itself being able to be fully covered by a single set. So then using that fact I could then use the rest of my argument to say that $Bbb N$ is not compact as given an open cover $B_alpha$ of $Bbb N$ then for each $nin Bbb N$ we can cover it with a set from T but we need infinitely many to do it for all points so it's not compact ?
    $endgroup$
    – can'tcauchy
    4 hours ago






  • 1




    $begingroup$
    I'm not sure what you mean, but you should use this definition of compactness. I've included an argument for why $mathbbN$ is not compact in my comment above that you can revisit after understanding the linked definition.
    $endgroup$
    – parsiad
    4 hours ago











  • $begingroup$
    @parsiad what you said in your comment is what I meant. Thanks ! But maybe the part I was unclear about was that I think I had been considering the set A=1,...n as being able to be covered by the set 1,....n in T, but as it says in the definition we have to think about A as being covered by points in a collection of open sets within A, and this can be reduced to a finite number of open sets which do this . Are the limit point and closure part of my argument is fine ,correct ?
    $endgroup$
    – can'tcauchy
    4 hours ago











  • $begingroup$
    Yes, I think that part of your argument is correct. I posted a detailed answer anyways.
    $endgroup$
    – parsiad
    4 hours ago














1












1








1





$begingroup$


Could someone tell me if my line of reasoning is correct here:



Say we have the topological space $(mathbbN, T)$ comprising of the empty set together with all subsets of $Bbb N$ that contain the element $1$.



I want to show that the closure of a compact set in this topology need not be compact.



Let $A equiv 1, ...., n$.
Then, this set is compact as it can be covered by a single open set in $T$.
Its limit points are every number in $Bbb N$ which is not 1.
Therefore, its closure is $overlineA = Bbb N$



If the above is true then I get confused because it seems that $Bbb N$ is an open set in $T$ so therefore can it not also be covered with a single open set in T and so would be compact as well? (I know intuitively that compact, being in some sense a measure of 'smallness', would indicate that $Bbb N$ shouldn't be compact but I don't see how to get that line of reasoning using the properties of this topology)










share|cite|improve this question











$endgroup$




Could someone tell me if my line of reasoning is correct here:



Say we have the topological space $(mathbbN, T)$ comprising of the empty set together with all subsets of $Bbb N$ that contain the element $1$.



I want to show that the closure of a compact set in this topology need not be compact.



Let $A equiv 1, ...., n$.
Then, this set is compact as it can be covered by a single open set in $T$.
Its limit points are every number in $Bbb N$ which is not 1.
Therefore, its closure is $overlineA = Bbb N$



If the above is true then I get confused because it seems that $Bbb N$ is an open set in $T$ so therefore can it not also be covered with a single open set in T and so would be compact as well? (I know intuitively that compact, being in some sense a measure of 'smallness', would indicate that $Bbb N$ shouldn't be compact but I don't see how to get that line of reasoning using the properties of this topology)







general-topology proof-writing compactness






share|cite|improve this question















share|cite|improve this question













share|cite|improve this question




share|cite|improve this question








edited 4 hours ago









Austin Mohr

20.8k35299




20.8k35299










asked 5 hours ago









can'tcauchycan'tcauchy

1,023417




1,023417







  • 3




    $begingroup$
    Your reasoning for $A$ being compact seems incorrect. $A = 1, ldots, n$ is compact because given an open cover $ B_alpha $ of $A$, we can find $B_alpha_k ni k$ for each $k in A$ so that $ B_alpha_k _k=1^n$ covers $A$. $mathbbN$ is not compact in your topology because the infinite cover $ 1, ldots, n _n=1^infty$ of $mathbbN$ cannot be reduce to a finite subcover.
    $endgroup$
    – parsiad
    4 hours ago











  • $begingroup$
    @parsiad Ah okay , so I should think of each one of the points in A needing to be covered by an open set in T rather than the set itself being able to be fully covered by a single set. So then using that fact I could then use the rest of my argument to say that $Bbb N$ is not compact as given an open cover $B_alpha$ of $Bbb N$ then for each $nin Bbb N$ we can cover it with a set from T but we need infinitely many to do it for all points so it's not compact ?
    $endgroup$
    – can'tcauchy
    4 hours ago






  • 1




    $begingroup$
    I'm not sure what you mean, but you should use this definition of compactness. I've included an argument for why $mathbbN$ is not compact in my comment above that you can revisit after understanding the linked definition.
    $endgroup$
    – parsiad
    4 hours ago











  • $begingroup$
    @parsiad what you said in your comment is what I meant. Thanks ! But maybe the part I was unclear about was that I think I had been considering the set A=1,...n as being able to be covered by the set 1,....n in T, but as it says in the definition we have to think about A as being covered by points in a collection of open sets within A, and this can be reduced to a finite number of open sets which do this . Are the limit point and closure part of my argument is fine ,correct ?
    $endgroup$
    – can'tcauchy
    4 hours ago











  • $begingroup$
    Yes, I think that part of your argument is correct. I posted a detailed answer anyways.
    $endgroup$
    – parsiad
    4 hours ago













  • 3




    $begingroup$
    Your reasoning for $A$ being compact seems incorrect. $A = 1, ldots, n$ is compact because given an open cover $ B_alpha $ of $A$, we can find $B_alpha_k ni k$ for each $k in A$ so that $ B_alpha_k _k=1^n$ covers $A$. $mathbbN$ is not compact in your topology because the infinite cover $ 1, ldots, n _n=1^infty$ of $mathbbN$ cannot be reduce to a finite subcover.
    $endgroup$
    – parsiad
    4 hours ago











  • $begingroup$
    @parsiad Ah okay , so I should think of each one of the points in A needing to be covered by an open set in T rather than the set itself being able to be fully covered by a single set. So then using that fact I could then use the rest of my argument to say that $Bbb N$ is not compact as given an open cover $B_alpha$ of $Bbb N$ then for each $nin Bbb N$ we can cover it with a set from T but we need infinitely many to do it for all points so it's not compact ?
    $endgroup$
    – can'tcauchy
    4 hours ago






  • 1




    $begingroup$
    I'm not sure what you mean, but you should use this definition of compactness. I've included an argument for why $mathbbN$ is not compact in my comment above that you can revisit after understanding the linked definition.
    $endgroup$
    – parsiad
    4 hours ago











  • $begingroup$
    @parsiad what you said in your comment is what I meant. Thanks ! But maybe the part I was unclear about was that I think I had been considering the set A=1,...n as being able to be covered by the set 1,....n in T, but as it says in the definition we have to think about A as being covered by points in a collection of open sets within A, and this can be reduced to a finite number of open sets which do this . Are the limit point and closure part of my argument is fine ,correct ?
    $endgroup$
    – can'tcauchy
    4 hours ago











  • $begingroup$
    Yes, I think that part of your argument is correct. I posted a detailed answer anyways.
    $endgroup$
    – parsiad
    4 hours ago








3




3




$begingroup$
Your reasoning for $A$ being compact seems incorrect. $A = 1, ldots, n$ is compact because given an open cover $ B_alpha $ of $A$, we can find $B_alpha_k ni k$ for each $k in A$ so that $ B_alpha_k _k=1^n$ covers $A$. $mathbbN$ is not compact in your topology because the infinite cover $ 1, ldots, n _n=1^infty$ of $mathbbN$ cannot be reduce to a finite subcover.
$endgroup$
– parsiad
4 hours ago





$begingroup$
Your reasoning for $A$ being compact seems incorrect. $A = 1, ldots, n$ is compact because given an open cover $ B_alpha $ of $A$, we can find $B_alpha_k ni k$ for each $k in A$ so that $ B_alpha_k _k=1^n$ covers $A$. $mathbbN$ is not compact in your topology because the infinite cover $ 1, ldots, n _n=1^infty$ of $mathbbN$ cannot be reduce to a finite subcover.
$endgroup$
– parsiad
4 hours ago













$begingroup$
@parsiad Ah okay , so I should think of each one of the points in A needing to be covered by an open set in T rather than the set itself being able to be fully covered by a single set. So then using that fact I could then use the rest of my argument to say that $Bbb N$ is not compact as given an open cover $B_alpha$ of $Bbb N$ then for each $nin Bbb N$ we can cover it with a set from T but we need infinitely many to do it for all points so it's not compact ?
$endgroup$
– can'tcauchy
4 hours ago




$begingroup$
@parsiad Ah okay , so I should think of each one of the points in A needing to be covered by an open set in T rather than the set itself being able to be fully covered by a single set. So then using that fact I could then use the rest of my argument to say that $Bbb N$ is not compact as given an open cover $B_alpha$ of $Bbb N$ then for each $nin Bbb N$ we can cover it with a set from T but we need infinitely many to do it for all points so it's not compact ?
$endgroup$
– can'tcauchy
4 hours ago




1




1




$begingroup$
I'm not sure what you mean, but you should use this definition of compactness. I've included an argument for why $mathbbN$ is not compact in my comment above that you can revisit after understanding the linked definition.
$endgroup$
– parsiad
4 hours ago





$begingroup$
I'm not sure what you mean, but you should use this definition of compactness. I've included an argument for why $mathbbN$ is not compact in my comment above that you can revisit after understanding the linked definition.
$endgroup$
– parsiad
4 hours ago













$begingroup$
@parsiad what you said in your comment is what I meant. Thanks ! But maybe the part I was unclear about was that I think I had been considering the set A=1,...n as being able to be covered by the set 1,....n in T, but as it says in the definition we have to think about A as being covered by points in a collection of open sets within A, and this can be reduced to a finite number of open sets which do this . Are the limit point and closure part of my argument is fine ,correct ?
$endgroup$
– can'tcauchy
4 hours ago





$begingroup$
@parsiad what you said in your comment is what I meant. Thanks ! But maybe the part I was unclear about was that I think I had been considering the set A=1,...n as being able to be covered by the set 1,....n in T, but as it says in the definition we have to think about A as being covered by points in a collection of open sets within A, and this can be reduced to a finite number of open sets which do this . Are the limit point and closure part of my argument is fine ,correct ?
$endgroup$
– can'tcauchy
4 hours ago













$begingroup$
Yes, I think that part of your argument is correct. I posted a detailed answer anyways.
$endgroup$
– parsiad
4 hours ago





$begingroup$
Yes, I think that part of your argument is correct. I posted a detailed answer anyways.
$endgroup$
– parsiad
4 hours ago











2 Answers
2






active

oldest

votes


















4












$begingroup$


Definition. Let $A subset X$ where $(X, tau)$ is a topological space.
We call $A$ compact if for each collection $ B_alpha subset tau$ such that $cup_alpha B_alpha supset A$, we can find a finite subcollection $B_alpha_1, ldots, B_alpha_n$ such that $B_alpha_1 cup cdots cup B_alpha_n supset A$.



Remark. We call the original collection an open cover and the subcollection a finite subcover.




Consider the topology in your original question.



Let $A equiv 1, ldots, n$ and $mathscrB equiv B_alpha$ be an open cover of $A$.
Let $k$ be a member of $A$.
Since $mathscrB$ is a cover of $A$, we can find $alpha_k$ such that $k in B_alpha_k$.
Therefore, $B_alpha_k_k=1^n$ covers $A$, and hence $A$ is compact.



Next, let $C_n equiv 1,ldots,n$ and consider the cover $mathscrC equiv C_n_n=1^infty$ of $mathbbN$.
Let $mathscrC^prime$ be a finite subcollection of $mathscrC$.
Note that the set $bigcup_C in mathscrC^prime C$ has a maximum element (call it $N$) and hence this subcollection does not cover $mathbbN$ (because none of its members contain $N+1$).
This shows that $mathbbN$ is not compact.



Next, let $n > 1$.
Since any open set in your topology must have 1 as a member, it follows that $n$ is a limit point of $A$.
Therefore, $overlineA = mathbbN$.



In summary, you have just found an example of a topology for which the closure of a compact set is not necessarily compact.
This is only possible for non-Hausdorff spaces.
Indeed, your topology is non-Hausdorff since any two non-empty neighbourhoods are not disjoint because they both contain the point 1.






share|cite|improve this answer









$endgroup$




















    1












    $begingroup$

    $A=1$ is compact as any cover of it has a one-element subcover.



    $overlineA = mathbb N$ which is not compact, as witnessed by the open cover $$1,2,1,3,1,4,ldots, 1,n, ldots$$ of $mathbb N$ from which we cannot omit a member (or it wouldn't cover), so has no finite subcover.






    share|cite|improve this answer









    $endgroup$













      Your Answer





      StackExchange.ifUsing("editor", function ()
      return StackExchange.using("mathjaxEditing", function ()
      StackExchange.MarkdownEditor.creationCallbacks.add(function (editor, postfix)
      StackExchange.mathjaxEditing.prepareWmdForMathJax(editor, postfix, [["$", "$"], ["\\(","\\)"]]);
      );
      );
      , "mathjax-editing");

      StackExchange.ready(function()
      var channelOptions =
      tags: "".split(" "),
      id: "69"
      ;
      initTagRenderer("".split(" "), "".split(" "), channelOptions);

      StackExchange.using("externalEditor", function()
      // Have to fire editor after snippets, if snippets enabled
      if (StackExchange.settings.snippets.snippetsEnabled)
      StackExchange.using("snippets", function()
      createEditor();
      );

      else
      createEditor();

      );

      function createEditor()
      StackExchange.prepareEditor(
      heartbeatType: 'answer',
      autoActivateHeartbeat: false,
      convertImagesToLinks: true,
      noModals: true,
      showLowRepImageUploadWarning: true,
      reputationToPostImages: 10,
      bindNavPrevention: true,
      postfix: "",
      imageUploader:
      brandingHtml: "Powered by u003ca class="icon-imgur-white" href="https://imgur.com/"u003eu003c/au003e",
      contentPolicyHtml: "User contributions licensed under u003ca href="https://creativecommons.org/licenses/by-sa/3.0/"u003ecc by-sa 3.0 with attribution requiredu003c/au003e u003ca href="https://stackoverflow.com/legal/content-policy"u003e(content policy)u003c/au003e",
      allowUrls: true
      ,
      noCode: true, onDemand: true,
      discardSelector: ".discard-answer"
      ,immediatelyShowMarkdownHelp:true
      );



      );













      draft saved

      draft discarded


















      StackExchange.ready(
      function ()
      StackExchange.openid.initPostLogin('.new-post-login', 'https%3a%2f%2fmath.stackexchange.com%2fquestions%2f3179012%2fshowing-the-closure-of-a-compact-subset-need-not-be-compact%23new-answer', 'question_page');

      );

      Post as a guest















      Required, but never shown

























      2 Answers
      2






      active

      oldest

      votes








      2 Answers
      2






      active

      oldest

      votes









      active

      oldest

      votes






      active

      oldest

      votes









      4












      $begingroup$


      Definition. Let $A subset X$ where $(X, tau)$ is a topological space.
      We call $A$ compact if for each collection $ B_alpha subset tau$ such that $cup_alpha B_alpha supset A$, we can find a finite subcollection $B_alpha_1, ldots, B_alpha_n$ such that $B_alpha_1 cup cdots cup B_alpha_n supset A$.



      Remark. We call the original collection an open cover and the subcollection a finite subcover.




      Consider the topology in your original question.



      Let $A equiv 1, ldots, n$ and $mathscrB equiv B_alpha$ be an open cover of $A$.
      Let $k$ be a member of $A$.
      Since $mathscrB$ is a cover of $A$, we can find $alpha_k$ such that $k in B_alpha_k$.
      Therefore, $B_alpha_k_k=1^n$ covers $A$, and hence $A$ is compact.



      Next, let $C_n equiv 1,ldots,n$ and consider the cover $mathscrC equiv C_n_n=1^infty$ of $mathbbN$.
      Let $mathscrC^prime$ be a finite subcollection of $mathscrC$.
      Note that the set $bigcup_C in mathscrC^prime C$ has a maximum element (call it $N$) and hence this subcollection does not cover $mathbbN$ (because none of its members contain $N+1$).
      This shows that $mathbbN$ is not compact.



      Next, let $n > 1$.
      Since any open set in your topology must have 1 as a member, it follows that $n$ is a limit point of $A$.
      Therefore, $overlineA = mathbbN$.



      In summary, you have just found an example of a topology for which the closure of a compact set is not necessarily compact.
      This is only possible for non-Hausdorff spaces.
      Indeed, your topology is non-Hausdorff since any two non-empty neighbourhoods are not disjoint because they both contain the point 1.






      share|cite|improve this answer









      $endgroup$

















        4












        $begingroup$


        Definition. Let $A subset X$ where $(X, tau)$ is a topological space.
        We call $A$ compact if for each collection $ B_alpha subset tau$ such that $cup_alpha B_alpha supset A$, we can find a finite subcollection $B_alpha_1, ldots, B_alpha_n$ such that $B_alpha_1 cup cdots cup B_alpha_n supset A$.



        Remark. We call the original collection an open cover and the subcollection a finite subcover.




        Consider the topology in your original question.



        Let $A equiv 1, ldots, n$ and $mathscrB equiv B_alpha$ be an open cover of $A$.
        Let $k$ be a member of $A$.
        Since $mathscrB$ is a cover of $A$, we can find $alpha_k$ such that $k in B_alpha_k$.
        Therefore, $B_alpha_k_k=1^n$ covers $A$, and hence $A$ is compact.



        Next, let $C_n equiv 1,ldots,n$ and consider the cover $mathscrC equiv C_n_n=1^infty$ of $mathbbN$.
        Let $mathscrC^prime$ be a finite subcollection of $mathscrC$.
        Note that the set $bigcup_C in mathscrC^prime C$ has a maximum element (call it $N$) and hence this subcollection does not cover $mathbbN$ (because none of its members contain $N+1$).
        This shows that $mathbbN$ is not compact.



        Next, let $n > 1$.
        Since any open set in your topology must have 1 as a member, it follows that $n$ is a limit point of $A$.
        Therefore, $overlineA = mathbbN$.



        In summary, you have just found an example of a topology for which the closure of a compact set is not necessarily compact.
        This is only possible for non-Hausdorff spaces.
        Indeed, your topology is non-Hausdorff since any two non-empty neighbourhoods are not disjoint because they both contain the point 1.






        share|cite|improve this answer









        $endgroup$















          4












          4








          4





          $begingroup$


          Definition. Let $A subset X$ where $(X, tau)$ is a topological space.
          We call $A$ compact if for each collection $ B_alpha subset tau$ such that $cup_alpha B_alpha supset A$, we can find a finite subcollection $B_alpha_1, ldots, B_alpha_n$ such that $B_alpha_1 cup cdots cup B_alpha_n supset A$.



          Remark. We call the original collection an open cover and the subcollection a finite subcover.




          Consider the topology in your original question.



          Let $A equiv 1, ldots, n$ and $mathscrB equiv B_alpha$ be an open cover of $A$.
          Let $k$ be a member of $A$.
          Since $mathscrB$ is a cover of $A$, we can find $alpha_k$ such that $k in B_alpha_k$.
          Therefore, $B_alpha_k_k=1^n$ covers $A$, and hence $A$ is compact.



          Next, let $C_n equiv 1,ldots,n$ and consider the cover $mathscrC equiv C_n_n=1^infty$ of $mathbbN$.
          Let $mathscrC^prime$ be a finite subcollection of $mathscrC$.
          Note that the set $bigcup_C in mathscrC^prime C$ has a maximum element (call it $N$) and hence this subcollection does not cover $mathbbN$ (because none of its members contain $N+1$).
          This shows that $mathbbN$ is not compact.



          Next, let $n > 1$.
          Since any open set in your topology must have 1 as a member, it follows that $n$ is a limit point of $A$.
          Therefore, $overlineA = mathbbN$.



          In summary, you have just found an example of a topology for which the closure of a compact set is not necessarily compact.
          This is only possible for non-Hausdorff spaces.
          Indeed, your topology is non-Hausdorff since any two non-empty neighbourhoods are not disjoint because they both contain the point 1.






          share|cite|improve this answer









          $endgroup$




          Definition. Let $A subset X$ where $(X, tau)$ is a topological space.
          We call $A$ compact if for each collection $ B_alpha subset tau$ such that $cup_alpha B_alpha supset A$, we can find a finite subcollection $B_alpha_1, ldots, B_alpha_n$ such that $B_alpha_1 cup cdots cup B_alpha_n supset A$.



          Remark. We call the original collection an open cover and the subcollection a finite subcover.




          Consider the topology in your original question.



          Let $A equiv 1, ldots, n$ and $mathscrB equiv B_alpha$ be an open cover of $A$.
          Let $k$ be a member of $A$.
          Since $mathscrB$ is a cover of $A$, we can find $alpha_k$ such that $k in B_alpha_k$.
          Therefore, $B_alpha_k_k=1^n$ covers $A$, and hence $A$ is compact.



          Next, let $C_n equiv 1,ldots,n$ and consider the cover $mathscrC equiv C_n_n=1^infty$ of $mathbbN$.
          Let $mathscrC^prime$ be a finite subcollection of $mathscrC$.
          Note that the set $bigcup_C in mathscrC^prime C$ has a maximum element (call it $N$) and hence this subcollection does not cover $mathbbN$ (because none of its members contain $N+1$).
          This shows that $mathbbN$ is not compact.



          Next, let $n > 1$.
          Since any open set in your topology must have 1 as a member, it follows that $n$ is a limit point of $A$.
          Therefore, $overlineA = mathbbN$.



          In summary, you have just found an example of a topology for which the closure of a compact set is not necessarily compact.
          This is only possible for non-Hausdorff spaces.
          Indeed, your topology is non-Hausdorff since any two non-empty neighbourhoods are not disjoint because they both contain the point 1.







          share|cite|improve this answer












          share|cite|improve this answer



          share|cite|improve this answer










          answered 4 hours ago









          parsiadparsiad

          18.7k32453




          18.7k32453





















              1












              $begingroup$

              $A=1$ is compact as any cover of it has a one-element subcover.



              $overlineA = mathbb N$ which is not compact, as witnessed by the open cover $$1,2,1,3,1,4,ldots, 1,n, ldots$$ of $mathbb N$ from which we cannot omit a member (or it wouldn't cover), so has no finite subcover.






              share|cite|improve this answer









              $endgroup$

















                1












                $begingroup$

                $A=1$ is compact as any cover of it has a one-element subcover.



                $overlineA = mathbb N$ which is not compact, as witnessed by the open cover $$1,2,1,3,1,4,ldots, 1,n, ldots$$ of $mathbb N$ from which we cannot omit a member (or it wouldn't cover), so has no finite subcover.






                share|cite|improve this answer









                $endgroup$















                  1












                  1








                  1





                  $begingroup$

                  $A=1$ is compact as any cover of it has a one-element subcover.



                  $overlineA = mathbb N$ which is not compact, as witnessed by the open cover $$1,2,1,3,1,4,ldots, 1,n, ldots$$ of $mathbb N$ from which we cannot omit a member (or it wouldn't cover), so has no finite subcover.






                  share|cite|improve this answer









                  $endgroup$



                  $A=1$ is compact as any cover of it has a one-element subcover.



                  $overlineA = mathbb N$ which is not compact, as witnessed by the open cover $$1,2,1,3,1,4,ldots, 1,n, ldots$$ of $mathbb N$ from which we cannot omit a member (or it wouldn't cover), so has no finite subcover.







                  share|cite|improve this answer












                  share|cite|improve this answer



                  share|cite|improve this answer










                  answered 1 hour ago









                  Henno BrandsmaHenno Brandsma

                  115k349125




                  115k349125



























                      draft saved

                      draft discarded
















































                      Thanks for contributing an answer to Mathematics Stack Exchange!


                      • Please be sure to answer the question. Provide details and share your research!

                      But avoid


                      • Asking for help, clarification, or responding to other answers.

                      • Making statements based on opinion; back them up with references or personal experience.

                      Use MathJax to format equations. MathJax reference.


                      To learn more, see our tips on writing great answers.




                      draft saved


                      draft discarded














                      StackExchange.ready(
                      function ()
                      StackExchange.openid.initPostLogin('.new-post-login', 'https%3a%2f%2fmath.stackexchange.com%2fquestions%2f3179012%2fshowing-the-closure-of-a-compact-subset-need-not-be-compact%23new-answer', 'question_page');

                      );

                      Post as a guest















                      Required, but never shown





















































                      Required, but never shown














                      Required, but never shown












                      Required, but never shown







                      Required, but never shown

































                      Required, but never shown














                      Required, but never shown












                      Required, but never shown







                      Required, but never shown







                      Popular posts from this blog

                      名間水力發電廠 目录 沿革 設施 鄰近設施 註釋 外部連結 导航菜单23°50′10″N 120°42′41″E / 23.83611°N 120.71139°E / 23.83611; 120.7113923°50′10″N 120°42′41″E / 23.83611°N 120.71139°E / 23.83611; 120.71139計畫概要原始内容臺灣第一座BOT 模式開發的水力發電廠-名間水力電廠名間水力發電廠 水利署首件BOT案原始内容《小檔案》名間電廠 首座BOT水力發電廠原始内容名間電廠BOT - 經濟部水利署中區水資源局

                      Prove that NP is closed under karp reduction?Space(n) not closed under Karp reductions - what about NTime(n)?Class P is closed under rotation?Prove or disprove that $NL$ is closed under polynomial many-one reductions$mathbfNC_2$ is closed under log-space reductionOn Karp reductionwhen can I know if a class (complexity) is closed under reduction (cook/karp)Check if class $PSPACE$ is closed under polyonomially space reductionIs NPSPACE also closed under polynomial-time reduction and under log-space reduction?Prove PSPACE is closed under complement?Prove PSPACE is closed under union?

                      Is my guitar’s action too high? Announcing the arrival of Valued Associate #679: Cesar Manara Planned maintenance scheduled April 23, 2019 at 23:30 UTC (7:30pm US/Eastern)Strings too stiff on a recently purchased acoustic guitar | Cort AD880CEIs the action of my guitar really high?Μy little finger is too weak to play guitarWith guitar, how long should I give my fingers to strengthen / callous?When playing a fret the guitar sounds mutedPlaying (Barre) chords up the guitar neckI think my guitar strings are wound too tight and I can't play barre chordsF barre chord on an SG guitarHow to find to the right strings of a barre chord by feel?High action on higher fret on my steel acoustic guitar